Mr. Hernandez combines 1 gallon of orange juice, 3 pints of pineapple juice, and 2 quarts of lemon-lime soda to make punch for a party. He allows 2 cups of punch for each guest. Will there be enough punch to serve all 14 guests? How much, if any, punch will be left

Answers

Answer 1

Answer:

1 . There will be enough cup of punch to serve all 14 guest

2. There will be 2 cups of punch left.

Step-by-step explanation:

Mr  Hernandez combines 1 gallon of orange juice, 3 pint of pineapple juice and 2 quarts of lemon-lime soda to make punch for a party.

This combination makes up the punch.  We have to convert to cups to ascertain if the punch will be enough to serve 14 guest which are entitle to 2 cups of punch. The 14 guest will consume a total of 28 cups of punch .

1 gallon of  orange juice = 16 cups of orange juice

1 pint = 2 cups

3 pints of pineapple =  6 cups

1 quarts = 4 cups

2 quarts of lemon-lime soda = 8 cups

When you combine all the cups together, the total punch will be = 16 + 6 + 8 = 30  cups of punch.

The guest will consume a total of 14 × 2 = 28 cups of punch. There will  be 30 - 28 = 2 cups of punch left.


Related Questions

what is this phrase in numerical expression

triple the sum of seventeen and ten

pls help

Answers

Answer: so you would do 17 plus ten which is 27 so triple 3 so your answer is 27^3

Step-by-step explanation:

sum so your adding plus you need the exponent which is times 3

Find the surface area of the pyramid to the nearest whole number.

Answers

Answer:

i think its 248m^2

Step-by-step explanation:

Points X( -6, -4), Y( -2, -8), Z( 2,-4) are the vertices of a right triangle. What is the area of XYZ? Round to the nearest tenth.

Answers

Answer:

16

Step-by-step explanation:

distance between point x and z is 8 units

distance between point z and y is 4 units

8 * 4 = 32

formula for area of a triangle (l * w)/2

plugin: (8 * 4)/2 = 16

How many boys are there in an introductory geology course if 360 students are enrolled and there are five boys to every seven girls?

Answers

Answer:150 branliest

Step-by-step explanation:

Answer:

150

Step-by-step explanation:

360/(5+7)=30

30*5=150

Rewrite the percentage in the sentence below as a decimal.
The 20 overseas investors own 7.2% of the business.

Answers

Answer:

0.072

Step-by-step explanation:

Answer:0.072

Step-by-step explanation:

7.2% = 7.2/100

7.2 ➗ 100=0.072

Subtracting by adding up 65-39

Answers

Answer:

26

Step-by-step explanation:

In order to subtract by adding up 65-39, we need to add -39 to the value of 65. This can be rewritten in this way;

65+(-39)

The equation above is similar to the one given because the product of a minus and a plus sign will still give us back a minus sign.

on solving;

65+(-39) = 26

Carpetland carpet installers incur an average cost of $300 for each carpet installed. Joan Chin, the firm’s vice president, proposes a new procedure for installations, which she hopes will be more efficient. Joan plans to run a trial and hopes that the results of a trial period will enable her to conclude with a level of significance of 0.05 that the new procedure reduces the average cost to install a carpet. If Joan's hypothesis test results in a Type II error, what would this mean?

Answers

Answer:

A Type II error happens when the null hypothesis failed to be rejected, although it is false and the alternative hypothesis is true.

In this context, would be that the test does not give enough evidence to support the claim that the new procedure reduces the average cost, although it really does reduces it.

The new procedure is effective, but the sample does not give enough evidence.

Step-by-step explanation:

If f(x)=9x+2, What is the value of the function when x=4?

If f(x)=9x+2, for what value of x is the value of the function 29?

Answers

Answer:

1) 38

2) 3

Step-by-step explanation:

f(x)=36+2

f(x)=38

29=9x+2

27=9x

x=3

Question: What is the value of the function at x=−2?

Answer: y=2

Step-by-step explanation: I took the test.

*THIS IS THE CORRECT ANSWER. PLEASE DON'T ANSWER 3. IT IS WRONG!*

Please help ASAP! I will mark Brainliest! Please answer CORRECTLY! No guessing!

Answers

Answer:

Option D: 5591.93

Step-by-step explanation:

The best way to understand this question is to apply the formula in an indirect manner;

P - Principle number, Starting Value

T - Time

I - Interest

Let us convert the interest into decimal form, such that 3.8% is shifted two decimal points to the right ⇒ 0.038. Now add 1 to this value to get ⇒ 1.038. By PEDMAS, you would first raise this value to the span of 3 years as such:

(1.038)^3 = 1.118386872.......

The final step would by to multiply the starting value (investment $) $ 5,000 by this continuing value of 1.118386872:

5,000(1.118386872) = 5591.93436 ⇒ Rounded to (About) 5591.93

Answer:D

Step-by-step explanation:

principal=p=$5000

Rate=r=3.8%

Time=n=3 years

amount=a

a=p(1+r/100)^n

a=5000(1+3.8/100)^3

a=5000(1+0.038)^3

a=5000(1.038)^3

a=5000 x 1.038 x 1.038 x 1.038

a=5591.93

a=$5591.93

A grocery store has an average sales of $8000 per day. The store introduced several advertising campaigns in order to increase sales. To determine whether or not the advertising campaigns have been effective in increasing sales, a sample of 64 days of sales was selected. It was found that the average was $8300 per day. From past information, it is known that the standard deviation of the population is $1200. The p-value is

Answers

Answer:

The p-value of the test is 0.023.

Step-by-step explanation:

In this case we need to determine whether the addition of several advertising campaigns increased the sales or not.

The hypothesis can be defined as follows:

H₀: The stores average sales is $8000 per day, i.e. μ = 8000.

Hₐ: The stores average sales is more than $8000 per day, i.e. μ > 8000.

The information provided is:

 [tex]n=64\\\bar x=\$8300\\\sigma=\$1200[/tex]

As the population standard deviation is provided, we will use a z-test for single mean.

Compute the test statistic value as follows:

 [tex]z=\frac{\bar x-\mu}{\sigma/\sqrt{n}}=\frac{8300-8000}{1200/\sqrt{64}}=2[/tex]

The test statistic value is 2.

Decision rule:

If the p-value of the test is less than the significance level then the null hypothesis will be rejected.

Compute the p-value for the two-tailed test as follows:

 [tex]p-value=P(Z>2)\\=1-P(Z<2)\\=1-0.97725\\=0.02275\\\approx 0.023[/tex]

*Use a z-table for the probability.

The p-value of the test is 0.023.

Writing Percents as Fractions
a. Write 35% as a fraction in simplest form.
35% =
35
100
Write as a fraction with
a denominator of 100.
7
20
Simplity.

Answers

Answer:

Answers are below

Step-by-step explanation:

Percent means "out of 100" so 35% is 35/100. We can simplify this fraction by dividing the numerator and denominator by 5. 35/100 / 5 = 7/20.

7/20 multiply the numerator and denominator by 5.

7/20 x 5 = 35/100

If this answer is correct, please make me Brainliest!

Answer:

b and c

Step-by-step explanation:

€1.4 m
1.9 m
Tim has to cover 3 tanks completely with paint.
Each tank is in the shape of a cylinder with a top and a bottom.
The tank has a diameter of 1.4 metres and a height of 1.9 m.
A tin of paint covers 5 m
Find the total surface area of the 3 tanks and state how
many tins of paint Tim will need to buy
You must show your working
Total marks 5

Answers

Answer:

Step-by-step explanation:

Since each tank is cylindrical, we would apply the formula for determining the total surface area of a cylinder which is expressed as

Total surface area = 2πr² + 2πrh

Where

r represents the height of the cylinder.

h represents the height of the cylinder.

π = 3.14

From the information given,

Height = 1.9

Diameter = 1.4

Radius = diameter/2 = 1.4/2 = 0.7

Total surface area = 2 × 3.14 × 0.7² + 2 × 3.14 × 0.7 × 1.9 = 3.0772 + 8.3524 = 11.4296 m²

Total surface area of the 3 cylinders = 11.4296 × 3 = 34.29 m²

Since a tin of paint covers 5 m², the number of tins paints that Tim will need to buy is

34.29/5 = 6.858

Since the tins of paint must be whole numbers, he needs to buy 7 tins of paints.

A bag contains 10 Yellow, 4 Green and 7 Blue marbles. Find the following probabilies.
P(blue)

Answers

Answer:

1/3

Step-by-step explanation:

The total number of marbles

10+4+7 = 21

P(blue) = blue marbles / total marbles

            = 7 / 21

            = 1/3

Please help ASAP! I will mark Brainliest! Please READ the question THEN answer CORRECTLY! No guessing!

Answers

Answer:

C.  [tex]\frac{\sqrt{5} }{8}[/tex]

Step-by-step explanation:

This expression can be rewritten as [tex]\frac{\sqrt{5} }{\sqrt{64} }[/tex].

Since the square root of 5 is prime and does not have any perfect square factors, it cannot be simplified. However, the square root of 64 is equal to 8, so our final simplified radical expression would be [tex]\frac{\sqrt{5} }{8}[/tex], which is option C.

HOPE THIS HELPED! :)

Answer:

C is the answer

Step-by-step explanation:

You have to rewrite as [tex]\sqrt{5}[/tex]/[tex]\sqrt{64}[/tex]. Then you have to simplify the denominator which is 8. That is how you get your answer.

Hope this helps.

The circle graph shows how Spencer spent his money in the month of July.

A circle graph representing Spencers expenses. 27 percent is clothing, 11 percent is Gasoline, 44 percent is Food and 18 percent is Entertainment.

Part A
If Spencer spent a total of $704.00 in the month of July, which is the best estimate for the amount of money he spent on clothing?
$70.00
$140.00
$210.00
$280.00
00:00

Part B
Which is the best estimate for the amount of money he spent on entertainment?
$140.00
$35.00
$105.00
$160.00

Answers

Answer:

$190.08

Step-by-step explanation:

Take 27% of the total $704 spent in July:

0.27($704) = $190.08

He spent an estimated $190.08 on clothing in July.

Answer:

Step-by-step explanation:

I THINK 140

A sphere is inscribed in a cube. Explain the relationship between the surface areas of the two solid figures

Answers

Answer:

No one knows. Jk just go look at the other brainly links. the question has already been asked and answered so you can go see them there.

Step-by-step explanation:

:)

Priscilla’s grandmother’s fruit salad recipe calls for one part apple, one part orange, four parts strawberry, two parts cherry, and three parts grape. Priscilla uses the same measuring cup to measure all of the fruit, so one part is equal to one cup of diced fruit. In this exercise, you will compare the quantities in the recipe to understand ratios better.

Answers

Answer:

There are 11 parts of fruit: 4-strawberry, 3- grape, 2, cherry, 1- apple, 1-orange 4/11 from the entine quantity of fruits- strawberry

3/11-grapes

2/11- cherries

1/11- apples

1/11- oranges } ⇒ the quantity of oranges and apples are equal Hope it helps ;>

Which fraction equals a repeating decimal?
30/50
13/25
5/30
13/10

Answers

Answer:

5/30

Step-by-step explanation:

5/30 = 1/6 =0.166666666...

The fraction equals a repeating decimal is 5/30.

What are decimals?

A decimal numeral system is the standard system for denoting integer and non-integer numbers. The way of denoting numbers in the decimal system is often referred to as decimal notation.

Now the given fractions are,

30/50

13/25

5/30

13/10

Converting them into decimals we get,

30/50 = 0.6

13/25 = 0.52

5/30 = 0.1666..

13/10 = 1.3

Thus the repeating decimal is 0.1666..

So, the fraction with repeating decimal is 5/30 = 0.1666..

Thus, the fraction equals a repeating decimal is 5/30.

To learn more about fraction :

brainly.com/question/1301963

#SPJ2

If you roll a die three times, what is the probability of rolling three ONES?

(Give your answer as a decimal, rounded to the nearest thousandth. That is, rounded to three decimal places.)

Answers

Answer:

0.004

Step-by-step explanation:

The probability of rolling any number is 1/6 so that times 3 is 1/216 that as a decimal i 0.004

Ms Davis is doing an activity with her statistic students where she gives them a 20 question multiple top choice test and they know none of the answers. Students need to guess on every question and each question has 5 possible choices, 1 of the which is correct.

What is the mean and standard deviation of the number of questions that each student gets correct?

Answers

Answer:

The mean and standard deviation of the number of questions that each student gets correct are 4 and 1.789 respectively.

Step-by-step explanation:

Let the random variable X be defined as the number of correct answers marked by a student.

It is provided that each question has 5 possible choices, 1 of the which is correct.

Then the probability of marking thee correct option is:

[tex]P(X)=\frac{1}{5}=0.20[/tex]

There are a total of n = 20 questions to be answered.

As the students does not the answer to any question, they would be guessing for each question. This implies that for a random question, all the five options has the equal probability of being correct and each of the five options can be correct independently from the other.

All these information above indicates that the random variable X follows a Binomial distribution with parameters n = 20 and p = 0.20.

The mean and standard deviation of a Binomial distribution are:

[tex]\mu=np\\\\\sigma=\sqrt{np(1-p)}[/tex]

Compute the mean and standard deviation of the random variable X as follows:

[tex]\mu=np=20\times 0.20=4\\\\\sigma=\sqrt{np(1-p)}=\sqrt{20\times 0.20\times(1-0.20)}=1.789[/tex]

Thus, the mean and standard deviation of the number of questions that each student gets correct are 4 and 1.789 respectively.

The mean will be "4" and the standard deviation will "1.789".

According to the question,

The probability of making 3 correct options will be:

→ [tex]P(X) = \frac{1}{5}[/tex]

            [tex]= 0.20[/tex]

Total number of questions,

n = 20

As we know,

The mean will be:

→ [tex]\mu = np[/tex]

By substituting the values, we get

     [tex]= 20\times 0.20[/tex]

     [tex]= 4[/tex]

and,

The standard deviation will be:

→ [tex]\sigma = \sqrt{np(1-p)}[/tex]

      [tex]= \sqrt{20\times 0.20\times (1-0.20)}[/tex]

      [tex]= 1.789[/tex]

Thus the responses above are correct.

Learn more about standard deviation here:

https://brainly.com/question/20896613

what is the value of the rational expression below when x is equal to 4? x+20/x+4

Answers

Answer:

3

Step-by-step explanation:

All we need to do here is plug in the number 4 for the variable x. Wherever there's an x, kidnap it and replace it with a 4!

[tex]\frac{x + 20}{x + 4}[/tex]

x = 4

[tex]\frac{4 + 20}{4 + 4}[/tex]

Do the addition.

4 + 20 = 24

4 + 4 = 8

[tex]\frac{24}{8}[/tex]

We can simplify this! What is 24 divided by 8?

24/8 = 3

The answer is 3!

What is the solution to the system of equations?

3 x + 10 y = negative 47. 5 x minus 7 y = 40.
(1, –5)
(1, 5)
(–1, –5)
(–1, 5)

Answers

Answer:

  (1, –5)

Step-by-step explanation:

It is relatively easy to try the offered solutions to see what works.

(1, -5)

  3(1) +10(-5) = -47 . . . true

  5(1) -7(-5) = 40 . . . true

(1, -5) is the solution

_____

As a check, you can try some of the other choices:

(1, 5)

  3(1) +10(5) ≠ -47

(-1, -5)

  3(-1) +10(-5) ≠ -47

(-1, 5)

  3(-1) +10(5) ≠ -47

None of the other choices works in the first equation, so they're not the solution.

Answer:

(1,-5)

Step-by-step explanation:

1)get desmos

2)put in numbers

3)where they intersect is the answer.

easy as pie... oh wait pie ain't easy...

easy as ramen

55 POINTS IF YOU GET RIGHT THEN GOATED AND BRAINIEST!!

Answers

Answer:

17 i would say it is D

18 i belive it is B

19 it is c

20 A

The management of Discount Furniture, a chain of discount furniture stores in the Northeast, designed an incentive plan for salespeople. To evaluate this innovative plan, 12 salespeople were selected at random, and their weekly incomes before and after the plan were recorded. Was there a significant increase in the typical salesperson’s weekly income due to the innovative incentive plan? Use the .05 significance level. Estimate the p-value, and interpret it

Answers

Answer:

Step-by-step explanation:

The question is incomplete. The complete question is

The management of Discount Furniture, a chain of discount furniture stores in the Northeast, designed an incentive plan for salespeople. To evaluate this innovative plan, 12 salespeople were selected at random, and their weekly incomes before and after the plan were recorded.

Salesperson Before After

Sid Mahone $320 $340

Carol Quick 290 285

Tom Jackson 421 475

Andy Jones 510 510

Jean Sloan 210 210

Jack Walker 402 500

Peg Mancuso 625 631

Anita Loma 560 560

John Cuso 360 365

Carl Utz 431 431

A. S. Kushner 506 525

Fern Lawton 505 619

Solution:

Corresponding income of salespersons before and after form matched pairs.

The data for the test are the differences between the income is salespersons.

μd = the​ income before minus their income after.

Bedore after diff

320 340 -20

290 285 5

421 475 - 54

510 510 0

210 210 0

402 500 - 98

625 631 -6

569 560 0

360 365 - 5

431 431 0

506 525 - 19

505 619 - 114

Sample mean, xd

= (- 20 + 5 - 54 + 0 + 0 - 98 - 6 + 0 - 5 + 0 + - 19 - 114)/12 = - 25.92

xd = - 25.92

Standard deviation = √(summation(x - mean)²/n

n = 12

Summation(x - mean)² = (- 20 + 25.92)^2 + (5 - 25.92)^2 + (- 54 + 25.92)^2+ (0 + 25.92)^2 + (0 + 25.92)^2 + ( - 98 + 25.92)^2 + ( - 6 + 25.92)^2 + (0 + 25.92)^2 + (- 5 + 25.92)^2 + (0 + 25.92)^2 + (- 19 + 25.92)^2 + (- 114 + 25.92)^2 = 17784.5168

Standard deviation = √(17784.5168/12

sd = 38.5

For the null hypothesis

H0: μd ≥ 0

For the alternative hypothesis

H1: μd < 0

1) The distribution is a students t. Therefore, degree of freedom, df = n - 1 = 12 - 1 = 11

2) The formula for determining the test statistic is

t = (xd - μd)/(sd/√n)

t = ( - 25.92- 0)/(38.5/√12)

t = - 2.33

3) We would determine the probability value by using the t test calculator.

p = 0.02

4) Assume alpha = 0.05

Since alpha, 0.05 > than the p value, 0.02, then we would reject the null hypothesis. We can conclude that at 5% significance level, there is a significant increase in the typical salesperson’s weekly income due to the innovative incentive plan

The sum of the measures of the angles of a triangle is 180. The sum of the measures of the second and third angles is five times the measure of the first angle. The third angle is 14 more than the second. Let x, y, and z represent the measures of the first, second, and third angles, respectively. Find the measures of the three angles.

Answers

Answer:

[tex]x=30\\y=68\\z=82[/tex]

Step-by-step explanation:

x = measure of the first angle

y = measure of the second angle

z = measure of the third angle

The sum of the measures of the second and third (y+z) is five times the measure of the first angle (=5x)

[tex]y+z=5x[/tex]

The third angle is 14 more than the second

[tex]z=y+14[/tex]

And remember that the sum of these three angles must be equal to 180.

[tex]x+y+z=180[/tex]

Let's take these equations

[tex]y+z=5x\\z=y+14\\x+y+z=180[/tex]

If you take a look at the first equation, we have y+z = 5x and we have y+z in the third equation as well, we can replace that....

[tex]x+y+z=180\\x+(y+z)=180\\x+(5x)=180[/tex]

Distribute the + sign

[tex]x+5x=180[/tex]

Combine like terms;

[tex]6x=180[/tex]

Divide by 6.

[tex]x=\frac{180}{6}\\ x=30[/tex]

We have now defined that the measure of the first angle is 30º.

Let's take another equation... for example [tex]z=y+14[/tex]

I'm going to take this one because if I replace x and z in the third equation, all I'll have left will be y.

[tex]x+y+z=180\\30+y+(y+14)=180[/tex]

Distribute the + sign and Combine like terms;

[tex]30+y+y+14=180\\44+2y=180\\[/tex]

Subtract 44 to isolate 2y.

[tex]2y=180-44\\2y=136[/tex]

Now divide by 2.

[tex]y=\frac{136}{2}\\ y=68[/tex]

We already have the value of x and y. Once again, replacing this in the third equation will leave us with z to solve for.

[tex]x+y+z=180\\30+68+z=180\\98+z=180\\z=180-98\\z=82[/tex]

What is the volume of a cylinder with a base radius of 4 and height 7?

Answers

Answer:

351.86

Step-by-step explanation:

If u(x)=-2x^2+3 and v(x)=1/x, what is the range of (uov)(x)

Answers

Answer:

Do my question i will do yours

Step-by-step explanation:

Answer:

The Answer is C

Step-by-step explanation:

Find the greatest common factor of the
following monomials:
34c3 2c5

Answers

Answer:2c^2

Step-by-step explanation:

34c^3 and 2c^5

34c^3=2 x 16 x c x c^2

2c^5=2 x c^2 x c^3

Greatest common factor =2 x c^2

Greatest common factor =2c^2

In the past, 19% of all homes with a stay-at-home parent had the father as the stay-at-home parent. An independent research firm has been charged with conducting a sample survey to obtain more current information. (a) What sample size is needed if the research firm's goal is to estimate the current proportion of homes with a stay-at-home parent in which the father is the stay-at-home parent with a margin of error of 0.03? Use a 95% confidence level. (Round your answer up to the nearest whole number.)

Answers

Answer:

A sample size of 657 is needed.

Step-by-step explanation:

In a sample with a number n of people surveyed with a probability of a success of [tex]\pi[/tex], and a confidence level of [tex]1-\alpha[/tex], we have the following confidence interval of proportions.

[tex]\pi \pm z\sqrt{\frac{\pi(1-\pi)}{n}}[/tex]

In which

z is the zscore that has a pvalue of [tex]1 - \frac{\alpha}{2}[/tex].

The margin of error is:

[tex]M = z\sqrt{\frac{\pi(1-\pi)}{n}}[/tex]

95% confidence level

So [tex]\alpha = 0.05[/tex], z is the value of Z that has a pvalue of [tex]1 - \frac{0.05}{2} = 0.975[/tex], so [tex]Z = 1.96[/tex].

In the past, 19% of all homes with a stay-at-home parent had the father as the stay-at-home parent.

This means that [tex]\pi = 0.19[/tex]

(a) What sample size is needed if the research firm's goal is to estimate the current proportion of homes with a stay-at-home parent in which the father is the stay-at-home parent with a margin of error of 0.03?

A sample size of n is needed.

n is found when [tex]M = 0.03[/tex]

Then

[tex]M = z\sqrt{\frac{\pi(1-\pi)}{n}}[/tex]

[tex]0.03 = 1.96\sqrt{\frac{0.19*0.81}{n}}[/tex]

[tex]0.03\sqrt{n} = 1.96\sqrt{0.19*0.81}[/tex]

[tex]\sqrt{n} = \frac{1.96\sqrt{0.19*0.81}}{0.03}[/tex]

[tex](\sqrt{n})^{2} = (\frac{1.96\sqrt{0.19*0.81}}{0.03})^{2}[/tex]

[tex]n = 656.91[/tex]

Rounding up to the nearest whole number.

A sample size of 657 is needed.

the midpoint of AB is point P at (-16,6) if point A is at (-10,8) what are the coordinates of point B?

Answers

Answer:

Denote B(x, y), we have:

-10 + x = 2 x (-16) => x = -22

  8 + y = 2 x 6 => y = 4

=> B(-22, 4)

Hope this helps!

:)

Other Questions
Find the area of the pentagon.A) 107 inches2 B) 116 inches2 C) 150 inches2 D) 182 inches2 Find the area of the pentagon.A) 107 inches2 B) 116 inches2 C) 150 inches2 D) 182 inches2 Write the most important five moral rules that shouldbe applied in a laboratory help me with this please Which is a FALSE statement about the blood that leaves the heart through the aorta?A.It is oxygenated.B.It contains high levels of sugar.C.It contains high levels of carbon dioxide.D.It is transported by the arteries to cells throughout the body. How didWorld War I contribute to revolution in Russia? Harry and Meghan have considered starting their own business but are concerned about the possibility of losing even their personal assets if the business fails. One way for BOTH Harry and Meghan to avoid this liability risk would be to : A.divorce as soon as possible and establish two sole proprietorships B.Organize a limited partnership with Harry as the general partner C.set up offshore accounts D.form a corporation A triangle has angles that measure 59, 48, and 73. What kind of triangle is it? Si Juan Candela hubiera tenido la posibilidad de estudiar sera reconocido en el mundo como: (don't translate it I know what it means)a)Un profesor universitariob)Un escritor de cuentos famososc)Un gran ingeniero d)Un periodista Two water waves meet at the same point, one having a displacement above equilibrium of 60 cm and the other having a displacement above equilibrium of 80 cm. what is the independent value The vertices of XYZ are X(4, 5), Y(4, 13), and Z(8, 9). Find an equation of the circle circumscribed about XYZ.An equation is ???NEEED HELPPP PLEASEEE Mike and Laura go to a restaurant for their first date. What does Mark do or say that makes Laura believe he has many positive character traits? Check all that apply. What is the concentration of a solution in which 10.0 g of AgNO, is dissolved in 450 mL of solution?M What are the pros and cons of having a guinea pig as a pet? Which fraction is equivalent to 15/30A 3/4B 2/4C 3/5D 5/6 Brittany Monroe is a legal secretary. Her biweekly salary is $1,650.00 what is her annual salary? Why might Lincoln be especially be concerned about maryland? What is it near "Only one remains." Ryan signals to his brother from his hiding place.Matt nods in acknowledgement, spotting the last evil robot."34 degrees." Matt signals back, informing Ryan of the angle he observed between Ryan and the robot.ColRyan records this value on his diagram (shown below) and performs a calculation. Calibrating his laser cannon tothe correct distance, he stands, aims, and fires.To what distance did Ryan calibrate his laser cannon?Do not round during your calculations. Round your final answer to the nearest meter.PromProRyanTea146 m120Matt34 (3/7)*(1/10)-(3/7)*2/5+3/7*1solve the above-using propeties of rational numbers..pls give the ans fast.. it's very urgent...pls don't give wrong ans... I will give you all five stars